Please confirm topic selection

Are you sure you want to trigger topic in your Anconeus AI algorithm?

Please confirm action

You are done for today with this topic.

Would you like to start learning session with this topic items scheduled for future?

Review Question - QID 100972

In scope icon M 1 E
QID 100972 (Type "100972" in App Search)
A 45-year-old man presents with a 3-day history of right-sided flank pain due to a lodged ureteral stone. What changes would be expected to be seen at the level of glomerular filtration?


Increase in glomerular capillary oncotic pressure

6%

8/140

Increase in Bowman's space capillary oncotic pressure

6%

9/140

Increase in Bowman's space hydrostatic pressure

77%

108/140

Increase in filtration fraction

2%

3/140

No change in filtration fraction

5%

7/140

Select Answer to see Preferred Response

bookmode logo Review TC In New Tab

Obstruction of the ureter, as can occur with utereolithiasis, causes an increase in pressure proximal to the obstruction that transmits back to the level of the glomerulus (Bowman's space) and can lead to hydronephrosis.

Obstruction of the distal urinary system results in a backflow of pressure to the glomerulus. This congestion causes an increased in bowman space hydrostatic pressure which opposes filtration and decreases the glomerular filtration rate (GFR). Filtration fraction is equal to the GFR/ renal plasma flow (RPF). RPF is a measure of renal perfusion and is unchanged with ureteral obstruction. The decreased GFR and unchanged RPF lead to a decreased filtration fraction.

The review article by Pietrow et al. discusses the medical management of urinary calculi, which can be prevented in most patients who have had an episode of stones by the use of calcium-sparing diuretics such as thiazides, if the underlying cause is hypercalciuria.

Gormley describes another cause of hydronephrosis, namely neurogenic bladder. This occurs via a similar mechanism in which a downstream occlusion of the urinary system leads to a backflow of pressure.

Illustration A displays a hydronephrotic kidney. Note the dilated renal pelvis and calyx.

Incorrect Answers:
1. Oncotic pressures are unaffected by ureteral obstruction.
2. Oncotic pressures are unaffected by ureteral obstruction.
4. Filtration fraction = GFR/RPF. Decreased GFR with an unchanged RPF causes a decrease in filtration fraction.
5. Filtration fraction = GFR/RPF. Decreased GFR with an unchanged RPF causes a decrease in filtration fraction.

ILLUSTRATIONS:
REFERENCES (2)
Authors
Rating
Please Rate Question Quality

3.5

  • star icon star icon star icon
  • star icon star icon star icon
  • star icon star icon star icon
  • star icon star icon star icon
  • star icon star icon star icon

(4)

Attach Treatment Poll
Treatment poll is required to gain more useful feedback from members.
Please enter Question Text
Please enter at least 2 unique options
Please enter at least 2 unique options
Please enter at least 2 unique options